Saturday, February 27, 2010

Heart Murmur Case 2

A 65-year-old man presents to his primary care physician complaining of dyspnea, chest pain, and several syncopal episodes. His symptoms have worsened over the past few months and his third syncopal episode prompted this visit. On examination, a systolic ejection murmur is
auscultated with an ejection click in the right second intercostal space. Rales are present at the lung bases. He has a history of rheumatic fever in his twenties.

Q 1
Which of the following is the most likely diagnosis?
/ A. Aortic regurgitation
/ B. Aortic stenosis
/ C. Mitral stenosis
/ D. Tension pneumothorax
/ E. Thoracic aortic dissection

Q 2
Patients with syncope cannot maintain sufficient cardiac output to meet peripheral perfusion demands. Which of the following best describes cardiac output?
/ A. Cardiac output = end diastolic volume - end systolic volume
/ B. Cardiac output = heart rate X mean arterial pressure
/ C. Cardiac output = heart rate X stroke volume
/ D. Cardiac output = stroke volume X mean arterial pressure
/ E. Cardiac output = systemic vascular resistance X mean arterial pressure

Q 3
Which of the following might explain the angina pectoris in this patient
/ A. Increased ventricular wall tension limits perfusion
/ B. Left ventricular hypertrophy accelerates atherosclerotic diseas
/ C. Pulmonary hypertension decreases the PO2 of arterial blood
/ D. Stenotic valves occlude the coronary arteries
/ E. Twisting of the heart on its axis limits coronary flow

Q 4
Which of the following sets of changes depict the mean arterial pressure (MAP), Ieft ventricular peak systolic pressure (LVPSP), pulmonary
wedge pressure (PWP), and left atrial pressure (LAP) in this patient, compared to a healthy individuaI?

____________________________________________________________________

Heart Murmur Case 2 Answers

A1
The correct answer is B. Exertional dyspnea, angina pectoris, and syncope are the cardinal symptoms of aortic stenosis. Exam findings of systolic ejection murmur with ejection click confirm this diagnosis. Patients experience dyspnea because of the pulmonary edema generated by increased pulmonary capillary pressure, transmitted from increased left heart pressures. Angina pectoris results from left ventricular hypertrophy and increased interventricular pressures. Thus, an increased myocardial mass increases myocardial oxygen demand, while the increased wall pressures decrease perfusion. Ischemia results. Syncope results when the impeded left ventricle cannot meet peripheral perfusion demands.
Patients would have a diastolic murmur with aortic regurgitation (choice A).
Patients will have fatigue and dyspnea with mitral stenosis (choice C), but they have a diastolic murmur.
Patients with tension pneumothorax (choice D) present emergently with acute onset of shortness of breath and hemodynamic instability.
Thoracic aortic dissection (choice E) presents as acute onset of "tearing" chest pain that radiates to the back. Murmur may also be present.

A2
The correct answer is C. Cardiac output = heart rate X stroke volume.
Choice A is incorrect: Stroke volume = end diastolic volume - end systolic volume
Choice B is incorrect: Double product (estimation of cardiac work) = mean arterial pressure X heart rate
Choices D and E are nonsense distracters.

A3
The correct answer is A. The angina pectoris seen in aortic stenosis is caused by left ventricular hypertrophy. The ventricle must generate greater pressures to overcome the occluded outflow tract, and hypertrophy occurs. This contributes to cardiac ischemia in several ways. As noted above, an increased myocardial mass increases myocardial oxygen demand, while the increased wall tension decreases perfusion. The myocardium is perfused during diastole, and coronary perfusion relies on this relaxation. When mural diastolic pressures remain elevated in hypertrophy, perfusion is limited and ischemia results.
Hypertrophy does not accelerate atherosclerosis (choice B).
Pulmonary hypertension (choice C), seen in aortic stenosis, results in pulmonary edema and a thus a mild hindrance to alveolar gas exchange. One would not expect the PO2 to decrease enough to cause angina.
Stenotic valves do not occlude the coronary arteries (choice D).
The heart does not twist to occlude the coronary arteries (choice E).

A4
The correct answer is A. In aortic stenosis, the blood is ejected from the left ventricle through a smaller than normal opening. Because the resistance to ejection of blood is high, the left ventricular peak systolic pressure can sometimes increase to over 250 mm Hg with normal pressures in the aorta. The increase in left ventricular pressure raises left atrial pressure as well as pulmonary wedge pressure (which is a clinical index of left atrial pressure).

No comments:

Post a Comment